Sr Examen

Otras calculadoras

9^x+(x-13)*3^x-9x+36=0 la ecuación

El profesor se sorprenderá mucho al ver tu solución correcta😉

v

Solución numérica:

Buscar la solución numérica en el intervalo [, ]

Solución

Ha introducido [src]
 x             x               
9  + (x - 13)*3  - 9*x + 36 = 0
$$\left(- 9 x + \left(3^{x} \left(x - 13\right) + 9^{x}\right)\right) + 36 = 0$$
Gráfica
Suma y producto de raíces [src]
suma
        -W(log(443426488243037769948249630619149892803)) + log(81)
1 + 2 + ----------------------------------------------------------
                                  log(3)                          
$$\frac{- W\left(\log{\left(443426488243037769948249630619149892803 \right)}\right) + \log{\left(81 \right)}}{\log{\left(3 \right)}} + \left(1 + 2\right)$$
=
    -W(log(443426488243037769948249630619149892803)) + log(81)
3 + ----------------------------------------------------------
                              log(3)                          
$$\frac{- W\left(\log{\left(443426488243037769948249630619149892803 \right)}\right) + \log{\left(81 \right)}}{\log{\left(3 \right)}} + 3$$
producto
  -W(log(443426488243037769948249630619149892803)) + log(81)
2*----------------------------------------------------------
                            log(3)                          
$$2 \frac{- W\left(\log{\left(443426488243037769948249630619149892803 \right)}\right) + \log{\left(81 \right)}}{\log{\left(3 \right)}}$$
=
2*(-W(log(443426488243037769948249630619149892803)) + log(81))
--------------------------------------------------------------
                            log(3)                            
$$\frac{2 \left(- W\left(\log{\left(443426488243037769948249630619149892803 \right)}\right) + \log{\left(81 \right)}\right)}{\log{\left(3 \right)}}$$
2*(-LambertW(log(443426488243037769948249630619149892803)) + log(81))/log(3)
Respuesta rápida [src]
x1 = 1
$$x_{1} = 1$$
x2 = 2
$$x_{2} = 2$$
     -W(log(443426488243037769948249630619149892803)) + log(81)
x3 = ----------------------------------------------------------
                               log(3)                          
$$x_{3} = \frac{- W\left(\log{\left(443426488243037769948249630619149892803 \right)}\right) + \log{\left(81 \right)}}{\log{\left(3 \right)}}$$
x3 = (-LambertW(log(443426488243037769948249630619149892803)) + log(81))/log(3)
Respuesta numérica [src]
x1 = 1.0
x2 = 2.0
x2 = 2.0